Determine if a market is an oligopoly

Assignment Help Microeconomics
Reference no: EM13188141

Managers of a monopoly under rate of return regulation have an incentive to

a. underestimate the firm's costs.

b. exaggerate the firm's costs.

c. minimize the monopoly's deadweight loss.

d. exaggerate the firm's profit.

e. earn zero economic profit.


Which of the following is correct?
i. All linear demand curves have a constant slope and a constant price elasticity of demand.
ii. The price elasticity of demand changes while moving along a downward-sloping linear demand curve.
iii. The magnitude of the slope of all linear demand curves is equal to the price elasticity of demand.

a. iii only.

b. ii only.

c. i, ii, and iii.

d. i only.

e. i and ii.


____ natural monopolies is a commonly used, potential solution to the problems presented by natural monopolies.

a. Giving incentives to firms to become

b. Regulating

c. Breaking up firms that are

d. Outlawing price discrimination by

e. Refusing to grant patents to

 

In the short run, a perfectly competitive firm

a. must suffer an economic loss.

b. must earn an economic profit.

c. might make an economic profit, incur an economic loss, or make a normal profit.

d. must make either an economic profit or a normal profit.

e. must earn a normal profit.

 

To determine if a market is an oligopoly, we need to determine if

a. cartels are legal in their market.

b. the firms are so few that they recognize their mutual interdependencies.

c. the firms make identical or differentiated products.

d. there are many firms in the market.

e. the market's HHI is less than 900.


Increasing marginal returns to labor

a. occur only when there are increasing marginal returns to capital.

b. are the result of specialization and division of labor in the production process.

c. describe the portion of a total product curve where the marginal product is negative.

d. mean that two workers produce less than twice the output of one worker.

e. occur when a particularly efficient worker is employed.

 

The return to entrepreneurship, which is what the firm's owner could earn running another business, is known as

a. entrepreneur's profit.

b. economic profit.

c. excessive profit.

d. accounting profit.

e. normal profit.

 

Both price supports and a price floor can

a. decrease the price below the equilibrium price.

b. increase consumer surplus.

c. create a deadweight loss.

d. decrease output below the equilibrium quantity.

e. have no effect on producer surplus.

 

In economics, cost is ________, and benefit is ________.

a. the amount of money that you pay for something; the amount of money that someone else is willing to pay you
b. what you must give up to get something; what you are willing to give up to get it
c. the amount of money that you pay on the margin; the amount of money that you receive on the margin
d. what you are willing to give up to get it; what you must give up to get something
e. what you are willing to pay on the margin; what the government pays you when you are unemployed or retired

If a substitute good is easy to find, then demand for a good is

a. unit elastic.

b. perfectly inelastic.

c. elastic.

d. inelastic.

e. Substitutes don't have any effect on elasticity.

Reference no: EM13188141

Questions Cloud

Explain what is the molecular weight of the polymer : A 40 foot fiberglass boat hull weighs 2000 kg. 60% of that is a fully cured crosslinked polyester resin (the remainder is the glass reinforcing fibers, pigment, fillers, etc.).
Implement a program that prompts the user for an integer : Implement a program that prompts the user for an integer n and prints the nth Fibonacci number, using the above algorithm.
What is the economic cost of jens business : oprah just inherited a house with a market value of 150,000 and she does not expect the market value to change. Each year, will pay 300 for utilities and 2000 in taxes. She can earn 4 percent on money in a bank account. Her cost of living in the h..
State how can one draw a best fit straight line : How can one draw a best fit straight line on a temperature versus time graph to determine the temperature at which two phases (solid plus melting point) appear
Determine if a market is an oligopoly : natural monopolies is a commonly used, potential solution to the problems presented by natural monopolies. a. Giving incentives to firms to become b. Regulating c. Breaking up firms that are d. Outlawing price discrimination by e. Refusing to grant p..
State mercury acetate is allowed to react completely : mercury(II) acetate is allowed to react completely with a solution containing 9.718 g of sodium sulfate, how many grams of the reactant in excess will remain after the reaction?
Create java servlets : Create a new JSON "view" (e.g., a JSP page that only displays the JSON representation of the data)
Define what is the molarity of sulfuric acid in the solution : What is the molarity of sulfuric acid in the solution? How long (in seconds) would it take to fill a 55 gallon drum, and how much sulfuric acid (lb_m) would the drum contain? (use two dimensional equations)?
What is the projected net income : Calculating Projected Net Income. A proposed new investment has projected sales of $825,000. Variable costs are 55 percent of sales, and fixed costs are $187,150; depreciation is $91,000. Prepare a pro forma income statement assuming a tax rate of..

Reviews

Write a Review

Microeconomics Questions & Answers

  What is maximized profit

The marginal cost of making a wedding cake is $200. In order to maximize profits, Laura should make more than 20 wedding cakes per month.  make fewer than 20 wedding cakes per month.

  What monetary policies do you think caused the crisis

What monetary policies do you think caused the crisis and what were the effects of the policies implemented in reaction to the crisis

  Write down the households budget constraints

Write down the household's budget constraints for period 1 and 2 and identify the current account.

  Perfectly competitive market

Assume that, in a perfectly competitive market at profit maximizing quantity, the market price is greater than average total cost. Describe what will happen to the number of companies,

  Would the worker continue to be a member of the labor

Assuming that U =0.5, would an hour spent in the labor force searching for a job make the husband better off or worse off? Would the worker continue to be a member of the labor force under these conditions? If not, how would he be ca..

  Computing maximum profit and revenue

Assume that instead of maximizing profit, the firm wants to maximize total revenue. Using algebra determine the optimal output, price, profit and revenue for the firm.

  Compare and contrast the theoretical implications

Critics of traditional welfare programs often argue that a downside of traditional welfare programs is that when the government gives lower income people money, it causes them to work less. Compare and contrast the theoretical implications on lab..

  Calculate the four-firm concentration ratio in the market

Ten firms compete in a market to sell product X. The total sales of all firms selling the product are $2 million. Ranking the firms' sales from highest to lowest, we find the top four firms' sales to be $260,000, $220,000, $150,000, and $130,000, ..

  Determination of optimal price and output

Two firms face the demand equation given by P=200,000 -6(Q1 + Q2) where Q1 and Q2 are the outputs of two firms. The total cost equations for two firms are given by: TC1 = 8000Q1 and TC2 = 8000Q2.

  The amendments to the constitution

Please avoid plagiarism, and write your words based on other people's facts. To get full points for this post, you need to write at least three facts about the part of the Constitution, and the why you think those facts shape the modern business..

  Explain marginal external costs

when the government increases taxes to provide traditional public goods, such national security, there tends to be what. marginal external cost equals marginal private cost minus marginal social cost.

  What is the expected value and variance of the number

What is the expected value and variance of the number of defective chips and what is the probability that they discover exactly 9 defective chips in this test?

Free Assignment Quote

Assured A++ Grade

Get guaranteed satisfaction & time on delivery in every assignment order you paid with us! We ensure premium quality solution document along with free turntin report!

All rights reserved! Copyrights ©2019-2020 ExpertsMind IT Educational Pvt Ltd